Research Group of Prof. Dr. M. Griebel
Institute for Numerical Simulation
maximize
next up previous contents
Next: Verschiebung Up: Eindeutigkeit Previous: Asymptotische Darstellung

Verschiebungssatz

 

  SATZ6575

OBdA nehmen wir tex2html_wrap_inline11429 an. Zunächst beweisen wir drei Lemmata.

  LEMMA6585

BEWEIS. Mit Hilfe der asymptotischen Darstellungen von tex2html_wrap_inline11507 und u' sieht man

eqnarray6591

tex2html_wrap_inline7963

  LEMMA6609

BEWEIS. Seien tex2html_wrap_inline11523 und tex2html_wrap_inline11525 die Konstanten aus Lemma 6.3. Setze

equation6616

Dieses Minimum existiert wegen tex2html_wrap_inline11527 für tex2html_wrap_inline11529 und tex2html_wrap_inline11531 . Zu tex2html_wrap_inline11533 wähle tex2html_wrap_inline11535 so, daß

  equation6622

ist. Dies ist möglich wegen tex2html_wrap_inline11537 für tex2html_wrap_inline11539 . Aus dem Satz über implizite Funktionen folgt, daß tex2html_wrap_inline11541 stetig gewählt werden kann. Aus Lemma 6.3 wissen wir bereits

equation6631

Die Ungleichung muß also noch für das Intervall tex2html_wrap_inline11543 bewiesen werden. Für ein tex2html_wrap_inline7801 aus diesem Intervall gilt wegen tex2html_wrap_inline11547 und (6.19)

equation6636

Wir verschieben tex2html_wrap_inline11549 um tex2html_wrap_inline11551 nach links. Lemma 6.3 impliziert

eqnarray6642

Wir haben nun gezeigt: tex2html_wrap_inline11553 für tex2html_wrap_inline11555 , tex2html_wrap_inline11557 . Setze tex2html_wrap_inline11559 , dann folgt die Behauptung. tex2html_wrap_inline7963

  LEMMA6647

BEWEIS. Analog zum Beweis von Lemma 6.3 gilt

eqnarray6654

tex2html_wrap_inline7963

Nun kann der Satz bewiesen werden:

BEWEIS. Lemma 6.4 und Lemma 6.5 zeigen: tex2html_wrap_inline11575 für tex2html_wrap_inline11577 , tex2html_wrap_inline11579 oder tex2html_wrap_inline11581 . Auf tex2html_wrap_inline11583 nimmt u' sein Maximum M<1 an. Durch hinreichend weites Verschieben von u nach links, d.h. hinreichend große Wahl von s, kann wegen tex2html_wrap_inline11593 für tex2html_wrap_inline11529 die Ungleichung tex2html_wrap_inline11597 erreicht werden. Also gilt dann tex2html_wrap_inline11575 . tex2html_wrap_inline7963



Marcel Arndt
Tue Mar 28 09:56:06 MSZ 2000